Line 25: Line 25:
 
Furthermore, let&nbsp;<span class="texhtml">''f''<sub>1</sub>(λ)</span>,&nbsp;<span class="texhtml">''f''<sub>2</sub>(λ)</span>, and&nbsp;<span class="texhtml">''f''<sub>3</sub>(λ)</span>&nbsp;be the spectral response functions for the three color outputs of a color camera. So for each pixel of the camera sensor, there is a 3-dimensional output vector given by&nbsp;<span class="texhtml">''F'' = [''F''<sub>1</sub>,''F''<sub>2</sub>,''F''<sub>3</sub>]<sup>''t''</sup></span>, where  
 
Furthermore, let&nbsp;<span class="texhtml">''f''<sub>1</sub>(λ)</span>,&nbsp;<span class="texhtml">''f''<sub>2</sub>(λ)</span>, and&nbsp;<span class="texhtml">''f''<sub>3</sub>(λ)</span>&nbsp;be the spectral response functions for the three color outputs of a color camera. So for each pixel of the camera sensor, there is a 3-dimensional output vector given by&nbsp;<span class="texhtml">''F'' = [''F''<sub>1</sub>,''F''<sub>2</sub>,''F''<sub>3</sub>]<sup>''t''</sup></span>, where  
  
<math>F_1 = \int_{-\infty}^{\infty}f_1(\lambda)I(\lambda)d\lambda</math>,
+
<math>F_1 = \int_{-\infty}^{\infty}f_1(\lambda)I(\lambda)d\lambda</math>,  
  
<math>F_2 = \int_{-\infty}^{\infty}f_2(\lambda)I(\lambda)d\lambda</math>,
+
<math>F_2 = \int_{-\infty}^{\infty}f_2(\lambda)I(\lambda)d\lambda</math>,  
  
<math>F_3 = \int_{-\infty}^{\infty}f_3(\lambda)I(\lambda)d\lambda</math>
+
<math>F_3 = \int_{-\infty}^{\infty}f_3(\lambda)I(\lambda)d\lambda</math>  
  
where&nbsp;<math>I(\lambda)</math>&nbsp;is the energy spectrum of the incoming light and&nbsp;<math>f_k(\lambda)\geq 0</math>&nbsp;for&nbsp;<math>k = 0, 1, 2.</math>.
+
where&nbsp;<span class="texhtml">''I''(λ)</span>&nbsp;is the energy spectrum of the incoming light and&nbsp;<math>f_k(\lambda)\geq 0</math>&nbsp;for&nbsp;<span class="texhtml">''k'' = 0,1,2.</span>.  
  
Furthermore, assume there exists a matrix,&nbsp;<math>M</math>, so that
+
Furthermore, assume there exists a matrix,&nbsp;<span class="texhtml">''M''</span>, so that  
  
 +
<br>
  
 +
a) Why is it necessary that&nbsp;<math>f_k(\lambda) \geq 0</math>&nbsp;for&nbsp;<span class="texhtml">''k'' = 0,1,2</span>?&lt;span style="line-height: 1.5em;" /&gt;
  
a) Why is it necessary that&nbsp;<math>f_k(\lambda) \geq 0</math>&nbsp;for&nbsp;<math>k = 0, 1, 2</math>?<span style="line-height: 1.5em;" />
+
b) Are the functions, <math> r_0(\lambda) \geq 0<math>, <math>g_0(\lambda) \geq 0<math>, and <math>b_0(\lambda) \geq 0<math>? If so, why? If not, why not?
  
b) Are the functions,&nbsp;
+
c) Derive an formula for the tristimulus vector <math>[r, g, b]^t <math> in terms of the tristimulus vector <math> F=[F_1, F_2, F_3]^t <math>.
  
 +
d) Do functions <math> f_k(\lambda) <math> exist, which meet these requirements? If so, give a specific example of such functions.
 
[[Category:ECE]] [[Category:QE]] [[Category:CNSIP]] [[Category:Problem_solving]] [[Category:Image_processing]]
 
[[Category:ECE]] [[Category:QE]] [[Category:CNSIP]] [[Category:Problem_solving]] [[Category:Image_processing]]

Revision as of 19:28, 10 November 2014


ECE Ph.D. Qualifying Exam

Communication, Networking, Signal and Image Processing (CS)

Question 5: Image Processing

August 2013



Question

Problem 1. (50 pts)


Problem 2. (50 pts)

Let r0(λ),  g0(λ) , and b0(λ) be the CIE color matching functions for red, green, and blue primaries at 700 nm, 546.1 nm, and 435.8 nm, respectively, and let [r,g,b] be the corresponding CIE tristimulus values. 

Furthermore, let f1(λ)f2(λ), and f3(λ) be the spectral response functions for the three color outputs of a color camera. So for each pixel of the camera sensor, there is a 3-dimensional output vector given by F = [F1,F2,F3]t, where

$ F_1 = \int_{-\infty}^{\infty}f_1(\lambda)I(\lambda)d\lambda $,

$ F_2 = \int_{-\infty}^{\infty}f_2(\lambda)I(\lambda)d\lambda $,

$ F_3 = \int_{-\infty}^{\infty}f_3(\lambda)I(\lambda)d\lambda $

where I(λ) is the energy spectrum of the incoming light and $ f_k(\lambda)\geq 0 $ for k = 0,1,2..

Furthermore, assume there exists a matrix, M, so that


a) Why is it necessary that $ f_k(\lambda) \geq 0 $ for k = 0,1,2?<span style="line-height: 1.5em;" />

b) Are the functions, $ r_0(\lambda) \geq 0<math>, <math>g_0(\lambda) \geq 0<math>, and <math>b_0(\lambda) \geq 0<math>? If so, why? If not, why not? c) Derive an formula for the tristimulus vector <math>[r, g, b]^t <math> in terms of the tristimulus vector <math> F=[F_1, F_2, F_3]^t <math>. d) Do functions <math> f_k(\lambda) <math> exist, which meet these requirements? If so, give a specific example of such functions. [[Category:ECE]] [[Category:QE]] [[Category:CNSIP]] [[Category:Problem_solving]] [[Category:Image_processing]] $

Alumni Liaison

Questions/answers with a recent ECE grad

Ryne Rayburn